Skip to main content

Optimal Structure of Experiential Services: Review and Extensions

  • Chapter
  • First Online:
Handbook of Service Science, Volume II

Abstract

In many consumer-intensive (B2C) services, delivering memorable customer experiences is often a source of competitive advantage. And yet, there exist few formal guidelines to design the structure of such experiences. In this chapter, we introduce a utility-based model of customer satisfaction when customers are subject to acclimation, satiation, and memory decay. We then review and extend principles for optimizing the structure of an experience to maximize customer satisfaction; specifically, we characterize the optimal sequence of activities, the optimal activity selection, and the optimal information policy about an uncertain outcome. We find that, in general, the optimal experience structure is non-monotone in service levels and makes use of breaks/intermissions to create contrasts and reset satiation levels. However, in many extreme cases, we show that a crescendo design is optimal. We then discuss the implications of our framework for quality management in services, especially as it relates to a potential gap between ex-ante expectation and ex-post satisfaction, and for monetizing customers’ utilities derived while anticipating or recalling the event.

This is a preview of subscription content, log in via an institution to check access.

Access this chapter

Chapter
USD 29.95
Price excludes VAT (USA)
  • Available as PDF
  • Read on any device
  • Instant download
  • Own it forever
eBook
USD 219.00
Price excludes VAT (USA)
  • Available as EPUB and PDF
  • Read on any device
  • Instant download
  • Own it forever
Softcover Book
USD 279.99
Price excludes VAT (USA)
  • Compact, lightweight edition
  • Dispatched in 3 to 5 business days
  • Free shipping worldwide - see info
Hardcover Book
USD 279.99
Price excludes VAT (USA)
  • Durable hardcover edition
  • Dispatched in 3 to 5 business days
  • Free shipping worldwide - see info

Tax calculation will be finalised at checkout

Purchases are for personal use only

Institutional subscriptions

Notes

  1. 1.

    Prospect theory posits that is concave for all x ≥ 0, convex for all x < 0, and exhibits loss aversion in the sense that −vk(−x) ≥ vk(x) ≥ 0 for all x > 0; see, e.g., Baucells and Sarin (2010) and Kőszegi and Rabin (2006).

  2. 2.

    Even without explicitly modeling primacy effects, we find that a U-shape sequence may be optimal under Model (6.5). See Proposition 5.

  3. 3.

    Baucells and Bellezza (2017) consider an even more general model with a discount factor that is period-specific and dependent on the magnitude of the utility experienced in that period.

  4. 4.

    A more general model with multi-attribute activities could consider that the intensity of each attribute moves proportionally to the budget allocated to the activity, i.e., consider attributes as rays specific to each activity.

  5. 5.

    Although a book writer has complete control over the unfolding of the story, a sports event or game manager may not fully control it; yet, the rules of the sport or game may be altered to induce more or less variance in outcomes, as is currently under consideration for the game of tennis (The Economist2017).

  6. 6.

    For instance, the trailer of the movie Morgan was compiled by IBM’s Watson; see https://www.ibm.com/blogs/think/2016/08/cognitive-movie-trailer/

References

  • Acemoglu, D., M. A. Dahleh, I. Lobel, A. Ozdaglar. 2011. Bayesian learning in social networks. The Review of Economic Studies 78(4) 1201-1236.

    Article  Google Scholar 

  • Aflaki, S., I. Popescu. 2013. Managing retention in service relationships. Management Science 60(2) 415-433.

    Article  Google Scholar 

  • Baucells, M., S. Bellezza. 2017. Temporal profiles of instant utility during anticipation, event, and recall. Management Science 63(3) 729-748.

    Article  Google Scholar 

  • Baucells, M., R. K. Sarin. 2007. Satiation in discounted utility. Operations Research 55(1) 170-181.

    Article  Google Scholar 

  • Baucells, M., R. K. Sarin 2010. Predicting utility under satiation and habit formation. Management Science 56(2) 286-301.

    Article  Google Scholar 

  • Baucells, M., D. Smith, M. Weber. 2016. Preferences over sequences: Empirical evidence from music. Working paper.

    Google Scholar 

  • Berry, L. L., L. P. Carbone, S. H. Haeckel. 2002. Managing the total customer experience. Sloan Management Review 43(3) 85-89.

    Google Scholar 

  • Bitner, M. J. 1992. Servicescapes: The impact of physical surroundings on customers and employees. The Journal of Marketing 56(2) 57-71.

    Article  Google Scholar 

  • Bleichrodt, H., U. Schmidt, H. Zank. 2009. Additive utility in prospect theory. Management Science 55(5) 863-873.

    Article  Google Scholar 

  • Braff, A., J. C. DeVine. 2008. Maintaining the customer experience. McKinsey Quarterly. December.

    Google Scholar 

  • Bellos, I., S. Kavadias. 2017. Service Design for a Holistic Customer Experience: A Process Perspective. Working Paper.

    Google Scholar 

  • Chase, R.B., S. Dasu. 2001. Want to perfect your company’s service? Use behavioral science. Harvard Business Review 79(6) 78-84.

    Google Scholar 

  • Das Gupta, A., U.S. Karmarkar, G. Roels. 2015. The Design of Experiential Services with Acclimation and Memory Decay: Optimal Sequence and Duration. Management Science 62(5) 1278-1296.

    Article  Google Scholar 

  • Dasu, S., R. B. Chase. 2010. Designing the soft side of customer service. MIT Sloan Management Review 52(1) 32-40.

    Google Scholar 

  • DeVine, J., K. Gilson. 2010. Using behavioral science to improve the customer experience. McKinsey Quarterly. February.

    Google Scholar 

  • Dixon, M. J., G. M. Thompson. 2016. Bundling and scheduling service packages with customer behavior: Model and heuristic. Production and Operations Management 25(1) 36-55.

    Article  Google Scholar 

  • Dixon, M., R. Verma. 2013. Sequence effects in service bundles: Implications for service design and scheduling. Journal of Operations Management 31(3) 138-152.

    Article  Google Scholar 

  • The Economist. 2017. The NextGen scoring system could help bring tennis into the 21st century. https://www.economist.com/blogs/gametheory/2017/05/modernising-tennis. Last accessed on October 26, 2017.

  • Ebbinghaus, H. 1913. Memory: A Contribution to Experimental Psychology. Teachers College, Columbia University, New York, NY. (Original work published in 1885).

    Book  Google Scholar 

  • Edgeworth, FY. 1881. Mathematical Psychics: An Essay on the Application of Mathematics to the Moral Sciences. C. Keagann Paul, London, UK. Reprinted in 1967 by M. Kelly, New York, NY.

    Google Scholar 

  • Ely, J., A. Frankel, E. Kamenica. 2015. Suspense and surprise. Journal of Political Economy 123(1) 215-260.

    Article  Google Scholar 

  • Frederickson, B. L., D. Kahneman. 1993. Duration neglect in retrospective evaluations of a affective episodes. Journal of Personality and Social Psychology 65(1) 45-55.

    Article  Google Scholar 

  • Gebbia, J. 2016. How Airbnb designs for trust. TED Talks. https://www.ted.com/talks/joe_gebbia_how_airbnb_designs_for_trust?language=en. Last accessed on February 6, 2016.

  • Haeckel, S. H., L. P. Carbone, L. L. Berry. 2003. How to lead the customer experience. Marketing Management, January/February, 18–23.

    Google Scholar 

  • Heskett, J. L., T. O. Jones, G.W. Loveman, W. E. Sasser Jr., L. A. Schlesinger. 1994. Putting the service-profit chain to work. Harvard Business Review 72(2) 164-174.

    Google Scholar 

  • Hsee, C.K., R.K. Abelson. 1991. Velocity relation: Satisfaction as a function of the first derivative of outcome over time. Journal of Personality and Social Psychology 60(3) 341-347.

    Article  Google Scholar 

  • Jevons, H. S. 1905. Essays in Economics. MacMillan, London.

    Google Scholar 

  • Kahneman, D., P.P. Wakker, R. Sarin. 1997. Back to Bentham? Explorations of experienced utility. The Quarterly Journal of Economics 112(2) 375-406.

    Article  Google Scholar 

  • Karmarkar, U. S., U. R. Karmarkar. 2014. Customer experience and service design. In Baglieri, E., Managing Consumer Services: Factory or Theater? Springer International Publishing 109-130.

    Google Scholar 

  • Karmarkar, U. S., G. Roels. 2015. An analytical framework for value co-production in services. Service Science 7(3) 163-180.

    Article  Google Scholar 

  • Kőszegi, B., M. Rabin. 2006. A model of reference-dependent preferences. The Quarterly Journal of Economics 121(4) 1133-1165.

    Google Scholar 

  • Koopmans, T. C. 1960. Stationary ordinal utility and impatience. Econometrica 28(2) 297-309.

    Article  Google Scholar 

  • Lemon, K. N., P. C. Verhoef. 2016. Understanding customer experience throughout the customer journey. Journal of Marketing 80(6) 69-96.

    Article  Google Scholar 

  • Lowenstein, G. 1987. Anticipation and the value of delayed consumption. Economic Journal 97(387) 666-684.

    Article  Google Scholar 

  • McKinsey. 2016. The CEO guide to customer experience. McKinsey Quarterly. August. http://www.mckinsey.com/business-functions/operations/our-insights/the-ceo-guide-to-customer-experience

  • Norton, M. I., D. Mochon, D. Ariely. 2012. The IKEA effect: When labor leads to love. Journal of Consumer Psychology 22(3) 453-460.

    Article  Google Scholar 

  • Oliver, R. L. 2015. Satisfaction: A Behavioral Perspective on the Consumer. Routledge, 2nd edition, Taylor and Francis, New York, NY.

    Google Scholar 

  • Ostrom, A. L., A. Parasuraman, D. E. Bowen, L. Patrício, C. A. Voss. 2015. Service research priorities in a rapidly changing context. Journal of Service Research 18(2) 127-159.

    Article  Google Scholar 

  • Parasuraman, A., V. A. Zeithaml, L. L. Berry. 1988. SERVQUAL: A multiple-item scale for measuring consumer perceptions of service quality. Journal of Retailing 64(1) 12-40.

    Google Scholar 

  • Patrício, L., R. P. Fisk, J. F. e. Cunha. 2008. Designing multi-interface service experiences: The service experience blueprint. Journal of Service Research 10(4) 318-334.

    Article  Google Scholar 

  • Patrício, L., R. P. Fisk, J. F. e. Cunha, L. Constantine. 2011. Multilevel service design: From customer Value constellation to service experience blueprint. Journal of Service Research 14(2) 180-200.

    Article  Google Scholar 

  • Patrício, L., A. Gustafsson, R. Fisk. 2018. Upframing service design and innovation for research impact. Journal of Service Research 21(1) 3-16.

    Article  Google Scholar 

  • Pine, B. J., J. H. Gilmore. 1998. Welcome to the experience economy. Harvard Business Review 76, 97-105.

    Google Scholar 

  • Plambeck, E. L., Q. Wang. 2013. Implications of hyperbolic discounting for optimal pricing and scheduling of unpleasant services that generate future benefits. Management Science 59(8) 1927-1946.

    Article  Google Scholar 

  • Pullman, M. E., M. A. Gross. 2004. Ability of experience design elements to elicit emotions and loyalty behaviors. Decision Sciences 35(3) 551-578.

    Article  Google Scholar 

  • Rahmani, M., G. Roels, U. S. Karmarkar. 2017. Contracting and work dynamics in collaborative projects. Production and Operations Management 26(4) 686-703.

    Article  Google Scholar 

  • Roels, G. 2014. Optimal design of coproductive services: Interaction and work allocation. Manufacturing & Service Operations Management 16(4) 578-594.

    Article  Google Scholar 

  • Roels, G., X. Su. 2013. Optimal design of social comparison effects: Setting reference groups and reference points. Management Science 60(3) 606-627.

    Article  Google Scholar 

  • Rust, R. T., R. W. Oliver. 2000. The real-time service product. In New Service Development: Creating Memorable Experiences, J. A. Fitzsimmons and M. J. Fitzsimmons, eds. Thousand Oaks, CA, Sage, 52-69.

    Chapter  Google Scholar 

  • Samuelson, P. 1937. A note on measurement of utility. The Review of Economic Studies 4 155-161.

    Article  Google Scholar 

  • Smith, J. E. 1995. Generalized Chebychev inequalities: theory and applications in decision analysis. Operations Research 43(5) 807-825.

    Article  Google Scholar 

  • Thaler, R. H. 2015. Misbehaving: The Making of Behavioral Economics. W.W. Norton & Company, New York, NY.

    Google Scholar 

  • Verhoef, P. C., K. N. Lemon, A. Parasuraman, A. Roggeveen, M. Tsiros, L. A. Schlesinger. 2009. Customer experience creation: Determinants, dynamics and management strategies. Journal of Retailing 85(1) 31-41.

    Article  Google Scholar 

  • Wathieu, L. 1997. Habits and the anomalies in intertemporal choice. Management Science 43(11) 1552-1563.

    Article  Google Scholar 

  • Zomerdijk, L. G., C. A. Voss. 2010. Service design for experience-centric services. Journal of Service Research 13(1) 67-82.

    Article  Google Scholar 

Download references

Acknowledgments

I would like to thank Uday S. Karmarkar, Lia Patrício, and two anonymous referees for sharing their insights and suggestions for improving the quality of the chapter.

Author information

Authors and Affiliations

Authors

Corresponding author

Correspondence to Guillaume Roels .

Editor information

Editors and Affiliations

Appendix

Appendix

Lemma 1:

For any p, q ≥ 1 and x ∈ [0, 1], 1 − xp − xq + xp − q ≥ 0.

Proof:

The derivative of the function 1 − xp − xq + xp − q with respect to p equals xp ln x (1 − xq) ≥ 0, and similarly for the derivative with respect to q. Hence, for any p, q ≥ 1, 1 − xp − xq + xp − q ≥ 1 − x−1 − x−1 + x−2 = (1 − x−1)2 ≥ 0.∎

Lemma 2:

For any p, q ≥ 1 and x ∈ [0, 1], 1 − xp − xq + xp − q is decreasing in x.

Proof:

The derivative of the function 1 − xp − xq + xp − q with respect to x equals \( \left(p+q\right)\ {x}^{-p-q-1}\left(\frac{p}{p+q}\ {x}^q+\frac{q}{p+q}\ {x}^p-1\right) \), and it is negative given that xq ≤ 1 and xp ≤ 1.∎

Proof of Proposition 1:

The proof uses an interchange argument. Throughout the proof, since K = 1, we omit the subscript k. Because α = γ = 0, \( S\left(\boldsymbol{x},{r}_1,{s}_1\right)={\sum}_{\tau =1}^T{\delta}^{T-\tau}\ v\left({x}_{\tau}-{r}_1\right)-v\left({s}_1\right) \). Suppose that, in the optimal sequence, Activity i starts in time period t and immediately precedes Activity j. In that case, because xτ = xi whenever πτ = i, we obtain

$$ {\displaystyle \begin{array}{c}S\left(\boldsymbol{x},{r}_1,{s}_1\right)=\sum \limits_{\tau =1}^{t-1}\ {\delta}^{T-\tau}\ v\left({x}_{\tau}-{r}_1\right)+\sum \limits_{\tau =t}^{t+{d}_i-1}\ {\delta}^{T-\tau}\ v\left({x}_i-{r}_1\right)\\ {}+\sum \limits_{\tau =t+{d}_i}^{t+{d}_i+{d}_j-1}\ {\delta}^{T-\tau}\ v\left({x}_j-{r}_1\right)\\ {}+\sum \limits_{\tau =t+{d}_i+{d}_j}^T\ {\delta}^{T-\tau}\ v\left({x}_{\tau}-{r}_1\right)-v\left({s}_1\right).\end{array}} $$

Consider a suboptimal sequence \( \tilde{\boldsymbol{x}} \) in which Activities i and j have been permuted. Because x is optimal, we must have that \( S\left(\boldsymbol{x},{r}_1,{s}_1\right)-S\left(\tilde{\boldsymbol{x}},{r}_1,{s}_1\right)\ge 0 \), i.e.,

$$ {\displaystyle \begin{array}{c}S\left(\boldsymbol{x},{r}_1,{s}_1\right)-S\left(\tilde{\boldsymbol{x}},{r}_1,{s}_1\right)\\ {}\kern2em =\sum \limits_{\tau =t}^{t+{d}_i-1}\ {\delta}^{T-\tau}\ v\left({x}_i-{r}_1\right)+\sum \limits_{\tau =t+{d}_i}^{t+{d}_i+{d}_j-1}\ {\delta}^{T-\tau}\ v\left({x}_j-{r}_1\right)\\ {}\kern2em -\sum \limits_{\tau =t}^{t+{d}_j-1}\ {\delta}^{T-\tau}\ v\left({x}_j-{r}_1\right)-\sum \limits_{\tau =t+{d}_j}^{t+{d}_i+{d}_j-1}\ {\delta}^{T-\tau}\ v\left({x}_i-{r}_1\right)\\ {}\kern2em =v\left({x}_i-{r}_1\right)\ \left(\sum \limits_{\tau =t}^{t+{d}_i-1}\ {\delta}^{T-\tau}-\sum \limits_{\tau =t+{d}_j}^{t+{d}_i+{d}_j-1}\ {\delta}^{T-\tau}\right)\\ {}\kern2em +v\left({x}_j-{r}_1\right)\left(\sum \limits_{\tau =t+{d}_i}^{t+{d}_i+{d}_j-1}\ {\delta}^{T-\tau}-\sum \limits_{\tau =t}^{t+{d}_j-1}\ {\delta}^{T-\tau}\right)\\ {}\kern2em =\left(v\left({x}_i-{r}_1\right)\right.\\ {}\kern2em \left.-v\left({x}_j-{r}_1\right)\right){\delta}^{T-t}\frac{1-{\delta}^{-{d}_i}-{\delta}^{-{d}_j}+{\delta}^{-{d}_i-{d}_j}}{1-{\delta}^{-1}}\ge 0,\end{array}} $$

which implies, by Lemma 1, that v(xi − r1) ≤ v(xj − r1). Because v(x) is increasing, this implies that xi ≤ xj. ∎

Proof of Proposition 2:

The proof uses an interchange argument. Throughout the proof, since K = 1, we omit the subscript k. Without loss of generality, we set r1 = 0. Because α = 0, \( S\left(\boldsymbol{x},{r}_1,{s}_1\right)={\sum}_{\tau =1}^T{\delta}^{T-\tau}\ \left(v\left({x}_{\tau}+{s}_{\tau}\right)-v\left({s}_{\tau}\right)\right) \). Suppose that, in the optimal sequence, Activity i starts in time period t and immediately precedes Activity j. In that case, because xτ = xi whenever πτ = i, we obtain

$$ {\displaystyle \begin{array}{c}S\left(\boldsymbol{x},{r}_1,{s}_1\right)=\sum \limits_{\tau =1}^{t-1}\ {\delta}^{T-\tau}\ \left(v\left({x}_{\tau}+{s}_{\tau}\right)-v\left({s}_{\tau}\right)\right)\\ {}+\sum \limits_{\tau =t}^{t+{d}_i-1}\ {\delta}^{T-\tau}\ \Big(v\left(\left(\tau -t+1\right)\ {x}_i+{s}_t\right)\\ {}-v\left(\left(\tau -t\right)\ {x}_i+{s}_t\right)\Big)\\ {}+\sum \limits_{\tau =t+{d}_i}^{t+{d}_i+{d}_j-1}\ {\delta}^{T-\tau}\Big(v\left(\left(\tau -t-{d}_i+1\right)\ {x}_j+{d}_i\ {x}_i+{s}_t\right)\\ {}-v\left(\left(\tau -t-{d}_i\right)\ {x}_j+{d}_i\ {x}_i+{s}_t\right)\Big)\\ {}+\sum \limits_{\tau =t+{d}_i+{d}_j}^T\ {\delta}^{T-\tau}\ \left(v\left({x}_{\tau}+{s}_{\tau}\right)-v\left({s}_{\tau}\right)\right)\\ {}=\sum \limits_{\tau =1}^{t-1}\ {\delta}^{T-\tau}\ \left(v\left({x}_{\tau}+{s}_{\tau}\right)-v\left({s}_{\tau}\right)\right)-v\left({s}_t\right)\ {\delta}^{T-t}\\ {}+\sum \limits_{\tau =t}^{t+{d}_i-1}\ {\delta}^{T-\tau -1}\ v\left(\left(\tau -t+1\right)\ {x}_i+{s}_t\right)\left(\delta -1\right)\\ {}+\sum \limits_{\tau =t+{d}_i}^{t+{d}_i+{d}_j-1}\ {\delta}^{T-\tau -1}\ v\Big(\left(\tau -t-{d}_i+1\right)\ {x}_j+{d}_i\ {x}_i\\ {}+{s}_t\Big)\left(\delta -1\right)+{\delta}^{T-t-{d}_i-{d}_j}\ v\left({d}_j\ {x}_j+{d}_i\ {x}_i+{s}_t\right)\\ {}+\sum \limits_{\tau =t+{d}_i+{d}_j+1}^T\ {\delta}^{T-\tau}\ \left(v\left({x}_{\tau}+{s}_{\tau}\right)-v\left({s}_{\tau}\right)\right).\end{array}} $$

Consider a suboptimal sequence \( \tilde{\boldsymbol{x}} \) in which Activities i and j have been permuted. Because x is optimal, we must have that \( S\left(\boldsymbol{x},{r}_1,{s}_1\right)-S\left(\tilde{\boldsymbol{x}},{r}_1,{s}_1\right)\ge 0 \). Because the terms associated with activities scheduled before t or after t + di + dj + 1 are identical across both expressions, we must thus have that

$$ {\displaystyle \begin{array}{c}S\left(\boldsymbol{x},{r}_1,{s}_1\right)-S\left(\tilde{\boldsymbol{x}},{r}_1,{s}_1\right)\\ {}\kern2em =\sum \limits_{\tau =t}^{t+{d}_i-1}{\delta}^{T-\tau -1}\ v\left(\left(\tau -t+1\right)\ {x}_i+{s}_t\right)\left(\delta -1\right)\\ {}\kern1em =\sum \limits_{\tau =t+{d}_i}^{t+{d}_i+{d}_j-1}{\delta}^{T-\tau -1}\ v\Big(\left(\tau -t-{d}_i+1\right)\ {x}_j+{d}_i\ {x}_i\\ {}\kern1em +{s}_t\Big)\left(\delta -1\right)\\ {}\kern1em -\sum \limits_{\tau =t}^{t+{d}_j-1}{\delta}^{T-\tau -1}\ v\left(\left(\tau -t+1\right)\ {x}_j+{s}_t\right)\left(\delta -1\right)\\ {}\kern1em -\sum \limits_{\tau =t+{d}_j}^{t+{d}_i+{d}_j-1}\ {\delta}^{T-\tau -1}\ v\Big(\left(\tau -t-{d}_j+1\right)\ {x}_i+{d}_j\ {x}_j\\ {}\kern1em +{s}_t\Big)\left(\delta -1\right)\ge 0,\end{array}} $$

which implies, since v(x) is increasing, that xi ≤ xj. ∎

Proof of Proposition 3:

The proof uses an interchange argument. Throughout the proof, since K = 1, we omit the subscript k. Because δ = 1 and γ = 0, \( S\left(\boldsymbol{x},{r}_1,{s}_1\right)={\sum}_{\tau =1}^Tv\left({x}_{\tau}-{r}_{\tau}\right)-v\left({s}_1\right) \). Suppose first that Activities i and j are not the last ones. Specifically, suppose that, in the optimal sequence, Activity i starts in time period t and immediately precedes Activity j, and that Activity j precedes Activity l. In that case, because xτ = xi whenever πτ = i, we obtain

$$ {\displaystyle \begin{array}{c}S\left(\boldsymbol{x},{r}_1,{s}_1\right)=\sum \limits_{\tau =1}^{t-1}\ v\left({x}_{\tau}-{r}_{\tau}\right)+v\left({x}_i-{r}_t\right)+v\left({x}_j-{x}_i\right)+v\left({x}_l-{x}_j\right)\\ {}\kern1em +\sum \limits_{\tau =t+{d}_i+{d}_j+{d}_l}^T\ v\left({x}_{\tau}-{r}_{\tau}\right)-v\left({s}_1\right).\end{array}} $$

Consider a suboptimal sequence \( \tilde{\boldsymbol{x}} \) in which Activities i and j have been permuted. Because x is optimal, we must have that \( S\left(\boldsymbol{x},{r}_1,{s}_1\right)-S\left(\tilde{\boldsymbol{x}},{r}_1,{s}_1\right)\ge 0 \). Because the terms associated with activities scheduled before t or after t + di + dj + dl are identical across both expressions, we obtain that

$$ {\displaystyle \begin{array}{c}S\left(\boldsymbol{x},{r}_1,{s}_1\right)-S\left(\tilde{\boldsymbol{x}},{r}_1,{s}_1\right)\\ {}\kern1em =v\left({x}_i-{r}_t\right)+v\left({x}_j-{x}_i\right)+v\left({x}_l-{x}_j\right)-v\left({x}_j-{r}_t\right)\\ {}\kern1em -v\left({x}_i-{x}_j\right)-v\left({x}_l-{x}_i\right)\ge 0.\end{array}} $$

We next show that this inequality holds if only if xi ≤ xj.Suppose first that xi ≤ xj. Because v(x) is subadditive, v(xj − rt) ≤ v(xj − xi) + v(xi − rt). Similarly, v(xl − xi) ≤ v(xl − xj) + v(xj − xi). Moreover, because of loss aversion, when xi ≤ xj, v(xj − xi) ≤  − v(xi − xj). Combining these inequalities yields the desired inequality. Conversely, suppose that xi > xj. Because v(x) is subadditive, v(xi − rt) ≤ v(xi − xj) + v(xj − rt). Similarly, v(xl − xj) ≤ v(xl − xi) + v(xi − xj). Moreover, because of loss aversion, when.xi > xj, v(xj − xi) <  − v(xi − xj). Combining these inequalities yields the opposite inequality. Hence, \( S\left(\boldsymbol{x},{r}_1,{s}_1\right)-S\left(\tilde{\boldsymbol{x}},{r}_1,{s}_1\right)\ge 0 \)implies that xi ≤ xj.

Next, suppose that Activities i and j are the last ones. Then,

$$ S\left(\boldsymbol{x},{r}_1,{s}_1\right)=\sum \limits_{\tau =1}^{t-1}\ v\left({x}_{\tau}-{r}_{\tau}\right)+v\left({x}_i-{r}_t\right)+v\left({x}_j-{x}_i\right)-v\left({s}_1\right). $$

Consider a suboptimal sequence \( \tilde{\boldsymbol{x}} \) in which Activities i and j have been permuted. Because x is optimal, we must have that

$$ {\displaystyle \begin{array}{c}S\left(\boldsymbol{x},{r}_1,{s}_1\right)-S\left(\tilde{\boldsymbol{x}},{r}_1,{s}_1\right)\\ {}\kern1em =v\left({x}_i-{r}_t\right)+v\left({x}_j-{x}_i\right)-v\left({x}_j-{r}_t\right)-v\left({x}_i-{x}_j\right)\\ {}\kern1em \ge 0.\end{array}} $$

Similar to the argument above for the case where Activities i and j are not the last ones, we can show that this inequality holds if and only if xi ≤ xj. Hence, \( S\left(\boldsymbol{x},{r}_1,{s}_1\right)-S\left(\tilde{\boldsymbol{x}},{r}_1,{s}_1\right)\ge 0 \)implies that xi ≤ xj.∎

Proof of Proposition 4:

The proof uses an interchange argument. Throughout the proof, since K = 1, we omit the subscript k. Because δ = γ = 1, S(x, r1, s1) = v(xT − rT + sT) − v(s1). Suppose first that Activities i and j are not the last ones. Specifically, suppose that, in the optimal sequence, Activity i starts in time period t and immediately precedes Activity j, and that Activity j precedes Activity l. In that case, because xτ = xi whenever πτ = i, we obtain

$$ {\displaystyle \begin{array}{c}{x}_T-{r}_T+{s}_T={x}_T-{r}_t+{s}_t+\sum \limits_{\tau =t}^{T-1}{\left(1-\alpha \right)}^{T-\tau}\ \left({x}_i-{r}_t\right)\\ {}+\sum \limits_{\tau =t+{d}_i}^{T-1}{\left(1-\alpha \right)}^{T-\tau}\ \left({x}_j-{x}_i\right)\\ {}+\sum \limits_{\tau =t+{d}_i+{d}_j}^{T-1}{\left(1-\alpha \right)}^{T-\tau}\ \left({x}_l-{x}_j\right)+\dots.\end{array}} $$

Consider a suboptimal sequence \( \tilde{\boldsymbol{x}} \) in which Activities i and j have been permuted. Because x is optimal, we must have that \( S\left(\boldsymbol{x},{r}_1,{s}_1\right)-S\left(\tilde{\boldsymbol{x}},{r}_1,{s}_1\right)\ge 0 \). Because the function v(x) is increasing, this implies that \( {x}_T-{r}_T+{s}_T\ge {\tilde{x}}_T-{\tilde{r}}_T+{\tilde{s}}_T \), in which \( {\tilde{r}}_T \) and \( {\tilde{s}}_T \) are the reference point and satiation level in period T corresponding to sequence \( \tilde{\boldsymbol{x}} \). Because the terms associated with activities scheduled before t or after t + di + dj + dl are identical across both expressions, as well as rt and xl, we must thus have that

$$ {\displaystyle \begin{array}{c}\left({x}_T-{r}_T+{s}_T\right)-\left({\tilde{x}}_T-{\tilde{r}}_T+{\tilde{s}}_T\right)\\ {}=\left({x}_i-{x}_j\right)\\ {}\times \Big(\sum \limits_{\tau =t}^{T-1}{\left(1-\alpha \right)}^{T-\tau}-\sum \limits_{\tau =t+{d}_i}^{T-1}{\left(1-\alpha \right)}^{T-\tau}\\ {}-\sum \limits_{\tau =t+{d}_j}^{T-1}{\left(1-\alpha \right)}^{T-\tau}+\sum \limits_{\tau =t+{d}_i+{d}_j}^{T-1}{\left(1-\alpha \right)}^{T-\tau}\Big)\ge 0.\end{array}} $$

After expanding the series, we obtain that

$$ {\displaystyle \begin{array}{c}\left({x}_i-{x}_j\right)\times \frac{{\left(1-\alpha \right)}^{T-t+1}}{\alpha}\\ {}\kern2em \times \left(-1+{\left(1-\alpha \right)}^{-{d}_i}+{\left(1-\alpha \right)}^{-{d}_j}-{\left(1-\alpha \right)}^{-{d}_i-{d}_j}\right)\ge 0\end{array}} $$

Using Lemma 1, we obtain that the second term in parentheses is always negative, which implies that xi ≤ xj.

When Activities i and j are the last two activities, we obtain, using a similar logic, that

$$ {\displaystyle \begin{array}{l}\left({x}_i-{x}_j\right)\times \left(\sum \limits_{\tau =t}^{T-1}{\left(1-\alpha \right)}^{T-\tau}-\sum \limits_{\tau =t+{d}_i}^{T-1}{\left(1-\alpha \right)}^{T-\tau}-\sum \limits_{\tau =t+{d}_j}^{T-1}{\left(1-\alpha \right)}^{T-\tau}\right)\\ {}\kern5em \ge 0.\end{array}} $$

After expanding the series using the fact that T − t = di + dj, we obtain

$$ {\displaystyle \begin{array}{c}\left({x}_i-{x}_j\right)\times \frac{{\left(1-\alpha \right)}^{d_i+{d}_j+1}}{\alpha}\\ {}\kern2em \times \left(-1+{\left(1-\alpha \right)}^{-{d}_i}+{\left(1-\alpha \right)}^{-{d}_j}-{\left(1-\alpha \right)}^{-{d}_i-{d}_j}\right)\ge 0\end{array}} $$

Using Lemma 1, we obtain that the second term in parentheses is always negative, which implies that xi ≤ xj. ∎

Proof of Proposition 5:

The proof uses an interchange argument. Because vk(x) = wk x for all k, \( S\left(\boldsymbol{x},{\boldsymbol{r}}_1,{\boldsymbol{s}}_1\right)={\sum}_{\tau =1}^T{\sum}_{k=1}^K{w}_k\ {\delta}^{T-\tau}\ \left({x}_{k,\tau}-{r}_{k,\tau}\right) \), i.e., the terms in sτ cancel each other. Suppose first that Activities i and j are not the last ones. Specifically, suppose that, in the optimal sequence, Activity i starts in time period t and immediately precedes Activity j, and that Activity j precedes Activity l. In that case, because xk, τ = xk, i whenever πτ = i, we obtain

$$ {\displaystyle \begin{array}{ll}S\left(\boldsymbol{x},{\boldsymbol{r}}_1,{\boldsymbol{s}}_1\right)=& \sum \limits_{\tau =1}^{t-1}\sum \limits_{k=1}^K{w}_k\ {\delta}^{T-\tau}\ \left({x}_{k,\tau}-{r}_{k,\tau}\right)\kern1em +\sum \limits_{\tau =t}^T\sum \limits_{k=1}^K{w}_k\ {\delta}^{T-\tau}{\left(1-\alpha \right)}^{\tau -t}\ \left({x}_{k,i}-{r}_{k,t}\right)\kern1em \\ {}& +\sum \limits_{\tau =t+{d}_i}^T\sum \limits_{k=1}^K{w}_k\ {\delta}^{T-\tau}{\left(1-\alpha \right)}^{\tau -t-{d}_i}\ \left({x}_{k,j}-{x}_{k,i}\right)\\ {}& +\sum \limits_{\tau =t+{d}_i+{d}_j}^T\sum \limits_{k=1}^K{w}_k\ {\delta}^{T-\tau}{\left(1-\alpha \right)}^{\tau -t-{d}_i-{d}_j}\ \left({x}_{k,l}-{x}_{k,j}\right)\\ {}& +\dots.\end{array}} $$

Consider a suboptimal sequence \( \tilde{\boldsymbol{x}} \) in which Activities i and j have been permuted. Because x is optimal, we must have that \( S\left(\boldsymbol{x},{\boldsymbol{r}}_1,{\boldsymbol{s}}_1\right)-S\left(\tilde{\boldsymbol{x}},{\boldsymbol{r}}_1,{\boldsymbol{s}}_1\right)\ge 0 \). Because the terms associated with activities scheduled before t or after t + di + dj + dl are identical across both expressions, as well as rk, t and xk, l, we thus obtain that

$$ {\displaystyle \begin{array}{c}S\left(\boldsymbol{x},{\boldsymbol{r}}_1,{\boldsymbol{s}}_1\right)-S\left(\tilde{\boldsymbol{x}},{\boldsymbol{r}}_1,{\boldsymbol{s}}_1\right)\\ {}\kern2em =\sum \limits_{\tau =t}^T\sum \limits_{k=1}^K{w}_k\ {\delta}^{T-\tau}{\left(1-\alpha \right)}^{\tau -t}\ \left({x}_{k,i}-{x}_{k,j}\right)\\ {}\kern2em +\sum \limits_{\tau =t+{d}_i}^T\sum \limits_{k=1}^K{w}_k\ {\delta}^{T-\tau}{\left(1-\alpha \right)}^{\tau -t-{d}_i}\ \left({x}_{k,j}-{x}_{k,i}\right)\\ {}\kern2em +\sum \limits_{\tau =t+{d}_j}^T\sum \limits_{k=1}^K{w}_k\ {\delta}^{T-\tau}{\left(1-\alpha \right)}^{\tau -t-{d}_j}\ \left({x}_{k,j}-{x}_{k,i}\right)\\ {}\kern2em +\sum \limits_{\tau =t+{d}_i+{d}_j}^T\sum \limits_{k=1}^K{w}_k\ {\delta}^{T-\tau}{\left(1-\alpha \right)}^{\tau -t-{d}_i-{d}_j}\ \left({x}_{k,i}-{x}_{k,j}\right)\\ {}\kern2em \ge 0.\end{array}} $$

Equivalently,

$$ {\displaystyle \begin{array}{l}\left(\sum \limits_{k=1}^K{w}_k\ \left({x}_{k,i}-{x}_{k,j}\right)\right)\\ {}\kern3em \times \Big(\sum \limits_{\tau =t}^T{\delta}^{T-\tau}{\left(1-\alpha \right)}^{\tau -t}-\sum \limits_{\tau =t+{d}_i}^T{\delta}^{T-\tau}{\left(1-\alpha \right)}^{\tau -t-{d}_i}\\ {}\kern3em -\sum \limits_{\tau =t+{d}_j}^T\ {\delta}^{T-\tau}{\left(1-\alpha \right)}^{\tau -t-{d}_j}\\ {}\kern3em +\sum \limits_{\tau =t+{d}_i+{d}_j}^T{\delta}^{T-\tau}{\left(1-\alpha \right)}^{\tau -t-{d}_i-{d}_j}\Big)\ge 0.\end{array}} $$

After expanding the series, we obtain

$$ {\displaystyle \begin{array}{l}\left(\sum \limits_{k=1}^K{w}_k\ \left({x}_{k,i}-{x}_{k,j}\right)\right)\times \frac{1}{\delta -\left(1-\alpha \right)}\\ {}\kern3em \times \left(-{\left(1-\alpha \right)}^{T-t+1}+{\left(1-\alpha \right)}^{T-t-{d}_i+1}\right)\\ {}\kern3em +{\left(1-\alpha \right)}^{T-t-{d}_j+1}-{\left(1-\alpha \right)}^{T-t-{d}_i-{d}_j+1}+{\delta}^{T-t+1}\\ {}\kern3em -{\delta}^{T-t-{d}_i+1}-{\delta}^{T-t-{d}_j+1}+{\delta}^{T-t-{d}_i-{d}_j+1}\Big)\ge 0.\end{array}} $$

Using Lemmas 1 and 2, that the last two terms are nonnegative if and only if \( t\le T+1-\frac{\ln \left(\frac{1-{\delta}^{-{d}_i}-{\delta}^{-{d}_j}+{\delta}^{-{d}_i-{d}_j}}{1-{\left(1-\alpha \right)}^{-{d}_i}-{\left(1-\alpha \right)}^{-{d}_j}+{\left(1-\alpha \right)}^{-{d}_i-{d}_j}}\right)}{\ln \left(\frac{1-\alpha}{\delta}\right)} \).

When Activities i and j are the last two activities, we obtain, using a similar logic, that

$$ {\displaystyle \begin{array}{l}\left(\sum \limits_{k=1}^K{w}_k\ \left({x}_{k,i}-{x}_{k,j}\right)\right)\\ {}\kern3em \times \Big(\sum \limits_{\tau =t}^T{\delta}^{T-\tau}{\left(1-\alpha \right)}^{\tau -t}-\sum \limits_{\tau =t+{d}_i}^T{\delta}^{T-\tau}{\left(1-\alpha \right)}^{\tau -t-{d}_i}\\ {}\kern3em -\sum \limits_{\tau =t+{d}_j}^T\ {\delta}^{T-\tau}{\left(1-\alpha \right)}^{\tau -t-{d}_j}\Big)\ge 0.\end{array}} $$

After expanding the series, it can be checked that the second term in parentheses is nonnegative if and only if \( t\ge T+1-\frac{\ln \left(\frac{1-{\delta}^{-{d}_i}-{\delta}^{-{d}_j}}{1-{\left(1-\alpha \right)}^{-{d}_i}-{\left(1-\alpha \right)}^{-{d}_j}}\right)}{\ln \left(\frac{1-\alpha}{\delta}\right)} \). Because, for any p, q ≥ 1, the function 1 − xp − xq is increasing in x, \( 1-{\delta}^{-{d}_i}-{\delta}^{-{d}_j}\ge 1-{\left(1-\alpha \right)}^{-{d}_i}-{\left(1-\alpha \right)}^{-{d}_j} \) if and only if δ ≥ 1 − α. Hence, the term \( \frac{\ln \left(\frac{1-{\delta}^{-{d}_i}-{\delta}^{-{d}_j}}{1-{\left(1-\alpha \right)}^{-{d}_i}-{\left(1-\alpha \right)}^{-{d}_j}}\right)}{\ln \left(\frac{1-\alpha}{\delta}\right)} \) is always negative, and therefore, \( t<T+1-\frac{\ln \left(\frac{1-{\delta}^{-{d}_i}-{\delta}^{-{d}_j}}{1-{\left(1-\alpha \right)}^{-{d}_i}-{\left(1-\alpha \right)}^{-{d}_j}}\right)}{\ln \left(\frac{1-\alpha}{\delta}\right)} \) for all Activities i and j. As a result, we must have that \( {\sum}_{k=1}^K{w}_k\ \left({x}_{k,i}-{x}_{k,j}\right)\le 0 \) if Activities i and j are the last ones and if Activity i precedes Activity j in the optimal sequence.∎

Proof of Proposition 6:

We first show by induction that \( {x}_{T-k}={r}_{T-k}+{\left({v}^{\hbox{'}}\right)}^{-1}\left(\frac{\lambda \left(1+ k\alpha \right)}{\delta^k}\right) \). Consider the Lagrangean function \( L\left(\left({x}_1,\dots, {x}_T\right),{r}_1,0\right)=S\left(\left({x}_1,\dots, {x}_T\right),{r}_1,0\right)-\lambda\ \left(\sum \limits_{t=1}^T{x}_t-B\right) \). Because the second-order optimality condition associated with xT, i.e., \( \frac{\partial^2L\left(\left({x}_1,\dots, {x}_T\right),{r}_1,0\right)}{\partial {x}_T^2}={v}^{{\prime\prime}}\left({x}_T-{r}_T\right)<0 \), is always satisfied by concavity of v(x), every stationary point defines a global maximum. Because xT = rT + (v)−1(λ) satisfies the first-order optimality condition associated with xT, i.e., \( \frac{\partial L\left(\left({x}_1,\dots, {x}_T\right),{r}_1,0\right)}{\partial {x}_T}={v}^{\prime}\left({x}_T-{r}_T\right)-\lambda =0 \), it is optimal to set \( {x}_T^{\ast}={r}_T+{\left({v}^{\hbox{'}}\right)}^{-1}\left(\lambda \right) \).

Fix k > 0 and suppose that it is optimal to set \( {x}_{T-l}^{\ast}={r}_{T-l}+{\left({v}^{\hbox{'}}\right)}^{-1}\left(\frac{\lambda \left(1+ l\alpha \right)}{\delta^l}\right) \) for l = 0, …, k − 1. Using the induction hypothesis, we obtain that

$$ {\displaystyle \begin{array}{l}\frac{\partial L\left(\left({x}_1,\dots, {x}_{T-k},{x}_{T-k+1}^{\ast},\dots, {x}_T^{\ast}\right),{r}_1,0\right)}{\partial {x}_{T-k}}\\ {}\kern5em ={\delta}^k\ {v}^{\prime}\left({x}_{T-k}-{r}_{T-k}\right)\\ {}\kern5em -\alpha \sum \limits_{l=0}^{k-1}{\delta}^l\ {\left(1-\alpha \right)}^{k-1-l}{v}^{\prime}\left({x}_{T-l}^{\ast}-{r}_{T-l}\right)-\lambda \\ {}\kern5em ={\delta}^k\ {v}^{\prime}\left({x}_{T-k}-{r}_{T-k}\right)-\alpha \sum \limits_{l=0}^{k-1}{\left(1-\alpha \right)}^{k-1-l}\left(\lambda \left(1+l\ \alpha \right)\right)\\ {}\kern5em -\lambda ={\delta}^k\ {v}^{\prime}\left({x}_{T-k}-{r}_{T-k}\right)-\lambda \left(1+k\ \alpha \right),\end{array}} $$

because

$$ {\displaystyle \begin{array}{l}\sum \limits_{l=0}^{k-1}{\left(1-\alpha \right)}^{-l}\left(1+l\ \alpha \right)\\ {}\kern5em =\frac{1-{\left(1-\alpha \right)}^{-k}}{1-{\left(1-\alpha \right)}^{-1}}\\ {}\kern5em +\alpha {\left(1-\alpha \right)}^{-1}\frac{1-k\ {\left(1-\alpha \right)}^{-\left(k-1\right)}+\left(k-1\right){\left(1-\alpha \right)}^{-k}}{{\left(1-{\left(1-\alpha \right)}^{-1}\right)}^2}\\ {}\kern5em =\frac{1-\alpha}{\alpha}\Big(-1+{\left(1-\alpha \right)}^{-k}+1-k\ {\left(1-\alpha \right)}^{-\left(k-1\right)}\\ {}\kern5em +\left(k-1\right){\left(1-\alpha \right)}^{-k}\Big)\\ {}\kern5em =\frac{1-\alpha}{\alpha}\ k\ {\left(1-\alpha \right)}^{-k}\ \left(-\left(1-\alpha \right)+1\ \right)=k\ {\left(1-\alpha \right)}^{1-k}.\end{array}} $$

Because \( \frac{\partial}{\partial {x}_{T-k}}\left(\frac{\partial L\left(\left({x}_1,\dots, {x}_{T-k},{x}_{T-k+1}^{\ast},\dots, {x}_T^{\ast}\right),{r}_1,0\right)}{\partial {x}_{T-k}}\right)={\delta}^k\ {v}^{{\prime\prime}}\left({x}_{T-k}-{r}_{T-k}\right)<0 \) and because \( {x}_{T-k}={r}_{T-k}+{\left({v}^{\hbox{'}}\right)}^{-1}\left(\frac{\lambda \left(1+ k\alpha \right)}{\delta^k}\right) \) solves \( \frac{\partial L\left(\left({x}_1,\dots, {x}_{T-k},{x}_{T-k+1}^{\ast},\dots, {x}_T^{\ast}\right),{r}_1,0\right)}{\partial {x}_{T-k}}=0 \), it is optimal to set \( {x}_{T-k}^{\ast}={r}_{T-k}+{\left({v}^{\hbox{'}}\right)}^{-1}\left(\frac{\lambda \left(1+ k\alpha \right)}{\delta^k}\right) \). This completes the induction step.

Because v(x) < 0, \( {x}_{T-k}^{\ast}-{r}_{T-k}={\left({v}^{\hbox{'}}\right)}^{-1}\left(\frac{\lambda \left(1+ k\alpha \right)}{\delta^k}\right) \) is decreasing in k.

Finally, suppose that v(x) = xβ for some 0 < β < 1 when x ≥ 0. In that case, \( {x}_{T-k}^{\ast}\ge {x}_{T-k-1}^{\ast} \) if and only if \( {r}_{T-k}+{\left(\frac{\lambda \left(1+ k\alpha \right)}{\beta\ {\delta}^k}\right)}^{\frac{1}{\beta -1}}\ge {x}_{T-k-1}^{\ast} \), i.e., if and only if \( {\left(\frac{\lambda \left(1+ k\alpha \right)}{\beta\ {\delta}^k}\right)}^{\frac{1}{\beta -1}}\ge \left(1-\alpha \right)\ {\left(\frac{\lambda \left(1+\left(k+1\right)\alpha \right)}{\beta\ {\delta}^{k+1}}\right)}^{\frac{1}{\beta -1}} \), i.e., if and only if \( \frac{\left(\frac{\lambda \left(1+ k\alpha \right)}{\beta\ {\delta}^k}\right)}{\left(\frac{\lambda \left(1+\left(k+1\right)\alpha \right)}{\beta\ {\delta}^{k+1}}\right)}\le {\left(1-\alpha \right)}^{\beta -1} \), i.e., if and only if \( \left(\frac{\delta\ \left(1+ k\alpha \right)}{\left(1+\left(k+1\right)\alpha \right)}\right)\le {\left(1-\alpha \right)}^{\beta -1} \). The left-hand side is increasing in k, whereas the right-hand side is constant, so there is at most one crossing. Because the left-hand side is equal to \( \left(\frac{\delta}{\left(1+\alpha \right)}\right) \) when k = 0 and to δ when k → ∞, and that both values are smaller than (1 − α)β − 1, we conclude that \( {x}_{T-k}^{\ast}\ge {x}_{T-k-1}^{\ast} \) for all k.∎

Proof of Proposition 7:

Without loss of generality, we set r1 = 0. Consider the Lagrangean function \( L\left(\left({x}_1,\dots, {x}_T\right),0,{s}_1\right)=S\left(\left({x}_1,\dots, {x}_T\right),0,{s}_1\right)-\lambda\ \left(\sum \limits_{t=1}^T{x}_t-B\right) \). Because the second-order optimality condition associated with xT, i.e., \( \frac{\partial^2L\left(\left({x}_1,\dots, {x}_T\right),0,{s}_1\right)}{\partial {x}_T^2}={v}^{{\prime\prime}}\left({x}_T+{s}_T\right)<0 \), is always satisfied by concavity of v(x), every stationary point defines a global maximum. Because xT =  − sT + (v)−1(λ) satisfies the first-order optimality condition associated with xT, i.e., \( \frac{\partial L\left(\left({x}_1,\dots, {x}_T\right),0,{s}_1\right)}{\partial {x}_T}={v}^{\prime}\left({x}_T+{s}_T\right)-\lambda =0 \), it is optimal to set \( {x}_T^{\ast}=-{s}_T+{\left({v}^{\hbox{'}}\right)}^{-1}\left(\lambda \right) \).

We next show by induction that \( {v}^{\prime}\left({x}_{T-k}^{\ast}+{s}_{T-k}\right)-\left(\gamma /\delta \right)\ {v}^{\hbox{'}}\left(\gamma\ \left({x}_{T-k}^{\ast}+{s}_{T-k}\right)\right)=\left(1-\gamma \right)\lambda /{\delta}^k \) for all k = 1, …, T − 1. Because

$$ {\displaystyle \begin{array}{l}\frac{\partial L\left(\left({x}_1,\dots, {x}_{T-1},{x}_T^{\ast}\right),0,{s}_1\right)}{\partial {x}_{T-1}}\\ {}\kern5em =\delta\ {v}^{\prime}\left({x}_{T-1}+{s}_{T-1}\right)+\gamma\ \left({v}^{\prime}\left({x}_T^{\ast}+{s}_T\right)-{v}^{\hbox{'}}\left({s}_T\right)\right)-\lambda \\ {}\kern5em =\delta\ {v}^{\prime}\left({x}_{T-1}+{s}_{T-1}\right)+\gamma\ \left(\lambda -{v}^{\prime}\left(\gamma\ \left({x}_{T-1}+{s}_{T-1}\right)\right)\right)\\ {}\kern5em -\lambda, \end{array}} $$

we obtain that

$$ {\displaystyle \begin{array}{l}\frac{\partial}{\partial {x}_{T-1}}\left(\frac{\partial L\left(\left({x}_1,\dots, {x}_{T-1},{x}_T^{\ast}\right),0,{s}_1\right)}{\partial {x}_{T-1}}\right)\\ {}\kern6em =\delta\ {v}^{{\prime\prime}}\left({x}_{T-1}+{s}_{T-1}\right)-{\gamma}^2\ {v}^{{\prime\prime}}\left(\gamma\ \left({x}_{T-1}+{s}_{T-1}\right)\right)<0,\end{array}} $$

by assumption, and it is thus optimal to set \( {x}_{T-1}^{\ast} \) such that \( \frac{\partial L\left(\left({x}_1,\dots, {x}_{T-1},{x}_T^{\ast}\right),0,{s}_1\right)}{\partial {x}_{T-1}}=\delta\ {v}^{\prime}\left({x}_{T-1}+{s}_{T-1}\right)+\gamma\ \left(\lambda -{v}^{\prime}\left(\gamma\ \left({x}_{T-1}+{s}_{T-1}\right)\right)\right)-\lambda =0 \).

Fix k > 0 and suppose that it is optimal to set \( {x}_{T-l}^{\ast} \) such that \( {v}^{\prime}\left({x}_{T-l}^{\ast}+{s}_{T-l}\right)-\left(\gamma /\delta \right)\ {v}^{\hbox{'}}\left(\gamma\ \left({x}_{T-l}^{\ast}+{s}_{T-l}\right)\right)=\left(1-\gamma \right)\lambda /{\delta}^l \) for all l = 1, …, k. Using the induction hypothesis, we obtain that

$$ {\displaystyle \begin{array}{l}\frac{\partial L\left(\left({x}_1,\dots, {x}_{T-k},{x}_{T-k+1}^{\ast},\dots, {x}_T^{\ast}\right),{r}_1,{s}_1\right)}{\partial {x}_{T-k}}\\ {}\kern5em ={\delta}^k\ {v}^{\prime}\left({x}_{T-k}+{s}_{T-k}\right)\\ {}\kern5em +\sum \limits_{l=0}^{k-1}{\delta}^l\ {\gamma}^{k-l}\ \left({v}^{\prime}\left({x}_{T-l}^{\ast}+{s}_{T-l}\right)-{v}^{\hbox{'}}\left({s}_{T-l}\right)\right)-\lambda \\ {}\kern5em ={\delta}^k\ {v}^{\prime}\left({x}_{T-k}+{s}_{T-k}\right)-{\delta}^{k-1}\ \gamma\ {v}^{\prime}\left({s}_{T-k+1}\right)\\ {}\kern5em +\sum \limits_{l=1}^{k-1}{\delta}^l\ {\gamma}^{k-l}\ \left({v}^{\prime}\left({x}_{T-l}^{\ast}+{s}_{T-l}\right)-\left(\frac{\gamma}{\delta}\right)\ {v}^{\prime}\left({s}_{T-l+1}\right)\right)\\ {}\kern5em +{\gamma}^k{v}^{\prime}\left({x}_T^{\ast}+{s}_T\right)-\lambda \\ {}\kern5em ={\delta}^k\ {v}^{\prime}\left({x}_{T-k}+{s}_{T-k}\right)-{\delta}^{k-1}\ \gamma\ {v}^{\prime}\left({s}_{T-k+1}\right)\\ {}\kern5em +\sum \limits_{l=1}^{k-1}\ {\gamma}^{k-l}\lambda\ \left(1-\gamma \right)+{\gamma}^k\lambda -\lambda \\ {}\kern5em ={\delta}^k\ {v}^{\prime}\left({x}_{T-k}+{s}_{T-k}\right)-{\delta}^{k-1}\ \gamma\ {v}^{\prime}\left({s}_{T-k+1}\right)+\gamma\ \lambda -\lambda.\end{array}} $$

Hence,

$$ {\displaystyle \begin{array}{l}\frac{\partial}{\partial {x}_{T-k}}\left(\frac{\partial L\left(\left({x}_1,\dots, {x}_{T-k},{x}_{T-k+1}^{\ast},\dots, {x}_T^{\ast}\right),{r}_1,{s}_1\right)}{\partial {x}_{T-1}}\right)\\ {}\kern7em ={\delta}^k\ {v}^{{\prime\prime}}\left({x}_{T-k}+{s}_{T-k}\right)-{\delta}^{k-1}\ {\gamma}^2\ v{\hbox{'}}^{\hbox{'}}\left(\gamma\ \left({x}_{T-k}+{s}_{T-k}\right)\right)\\ {}\kern7em <0,\end{array}} $$

and it therefore is optimal to set \( {x}_{T-k}^{\ast} \) such that \( \frac{\partial L\left(\left({x}_1,\dots, {x}_{T-k},{x}_{T-k+1}^{\ast},\dots, {x}_T^{\ast}\right),{r}_1,{s}_1\right)}{\partial {x}_{T-k}}=0 \). This completes the induction step.

Because \( {v}^{\prime}\left({x}_{T-k}^{\ast}+{s}_{T-k}\right)-\left(\frac{\gamma}{\delta}\right)\ {v}^{\hbox{'}}\left(\gamma\ \left({x}_{T-k}^{\ast}+{s}_{T-k}\right)\right)=\frac{\left(1-\gamma \right)\lambda}{\delta^k}<\frac{\left(1-\gamma \right)\lambda}{\delta^{k+1}}={v}^{\prime}\left({x}_{T-k-1}^{\ast}+{s}_{T-k-1}\right)-\left(\frac{\gamma}{\delta}\right)\ {v}^{\hbox{'}}\left(\gamma\ \left({x}_{T-k-1}^{\ast}+{s}_{T-k-1}\right)\right) \) and because the function \( {v}^{\prime}(x)-\left(\frac{\gamma}{\delta}\right)\ {v}^{\hbox{'}}\left(\gamma\ x\right) \) is decreasing by assumption, we obtain that \( {x}_{T-k}^{\ast}+{s}_{T-k}\ge {x}_{T-k-1}^{\ast}+{s}_{T-k-1} \) for any k = 1, …, T − 1. Moreover, because \( \left(1-\gamma \right){v}^{\prime}\left({x}_T^{\ast}+{s}_T\right)=\left(1-\gamma \right)\lambda =\delta\ {v}^{\hbox{'}}\left({x}_{T-1}^{\ast}+{s}_{T-1}\right)-\gamma\ {v}^{\hbox{'}}\left(\gamma\ \left({x}_{T-1}^{\ast}+{s}_{T-1}\right)\right)<\left(\delta -\gamma \right)\ {v}^{\hbox{'}}\left({x}_{T-1}^{\ast}+{s}_{T-1}\right)\le \left(1-\gamma \right)\ {v}^{\hbox{'}}\left({x}_{T-1}^{\ast}+{s}_{T-1}\right) \), since v(x) > 0, v’’(x) > 0, and δ ≤ 1; therefore, \( {x}_T^{\ast}+{s}_T>{x}_{T-1}^{\ast}+{s}_{T-1} \).

Finally, suppose that v(x) = xβ for some lnδ/ ln γ < β < 1 when x ≥ 0. In that case, \( {x}_{T-k}^{\ast}=-{s}_{T-k}+{\left(\frac{\delta^{k-1}\ \beta\ \left(\delta -{\gamma}^{\beta}\right)}{\lambda\ \left(1-\gamma \right)}\right)}^{\frac{1}{1-\beta}} \) for k = 1, …, T − 1 and \( {x}_T^{\ast}=-{s}_T+{\left(\frac{\beta}{\lambda}\right)}^{\frac{1}{1-\beta}} \). Suppose that T ≥ 3. Then, \( {x}_T^{\ast}\ge {x}_{T-1}^{\ast} \) if and only if \( {\left(\frac{\beta}{\lambda}\right)}^{\frac{1}{1-\beta}}\ge {x}_{T-1}^{\ast}+{s}_T={x}_{T-1}^{\ast}+\gamma \left({x}_{T-1}^{\ast}+{s}_{T-1}\right)=\left(1+\gamma \right)\left({x}_{T-1}^{\ast}+{s}_{T-1}\right)-{s}_{T-1}=\left(1+\gamma \right)\left({x}_{T-1}^{\ast}+{s}_{T-1}\right)-\gamma \left({x}_{T-2}^{\ast}+{s}_{T-2}\right)=\left(1+\gamma \right){\left(\frac{\beta\ \left(\delta -{\gamma}^{\beta}\right)}{\lambda\ \left(1-\gamma \right)}\right)}^{\frac{1}{1-\beta}}-\gamma {\left(\frac{\delta\ \beta\ \left(\delta -{\gamma}^{\beta}\right)}{\lambda\ \left(1-\gamma \right)}\right)}^{\frac{1}{1-\beta}} \). Hence, \( {x}_T^{\ast}\ge {x}_{T-1}^{\ast} \) if and only if \( 1\ge \left(\left(1+\gamma \right)-{\gamma \delta}^{\frac{1}{1-\beta}}\right){\left(\frac{\left(\delta -{\gamma}^{\beta}\right)}{\left(1-\gamma \right)}\right)}^{\frac{1}{1-\beta}} \), which is always true since \( \left(\left(1+\gamma \right)-{\gamma \delta}^{\frac{1}{1-\beta}}\right){\left(\frac{\left(\delta -{\gamma}^{\beta}\right)}{\left(1-\gamma \right)}\right)}^{\frac{1}{1-\beta}}\le \left(\left(1+\gamma \right)-\gamma \right){\left(\frac{\left(1-{\gamma}^{\beta}\right)}{\left(1-\gamma \right)}\right)}^{\frac{1}{1-\beta}}\le 1 \). Similarly, suppose that T − k ≥ 3. Then, \( {x}_{T-k}^{\ast}\ge {x}_{T-k-1}^{\ast} \) if and only if \( {\left(\frac{\delta^{k-1}\ \beta\ \left(\delta -{\gamma}^{\beta}\right)}{\lambda\ \left(1-\gamma \right)}\right)}^{\frac{1}{1-\beta}}\ge {x}_{T-k-1}^{\ast}+{s}_{T-k}={x}_{T-k-1}^{\ast}+\gamma \left({x}_{T-k-1}^{\ast}+{s}_{T-k-1}\right)=\left(1+\gamma \right)\left({x}_{T-k-1}^{\ast}+{s}_{T-k-1}\right)-{s}_{T-k-1}=\left(1+\gamma \right)\left({x}_{T-k-1}^{\ast}+{s}_{T-k-1}\right)-\gamma \left({x}_{T-k-2}^{\ast}+{s}_{T-k-2}\right)=\left(1+\gamma \right){\left(\frac{\delta^k\ \beta\ \left(\delta -{\gamma}^{\beta}\right)}{\lambda\ \left(1-\gamma \right)}\right)}^{\frac{1}{1-\beta}}-\gamma {\left(\frac{\delta^{k+1}\ \beta\ \left(\delta -{\gamma}^{\beta}\right)}{\lambda\ \left(1-\gamma \right)}\right)}^{\frac{1}{1-\beta}} \). Hence, \( {x}_{T-k}^{\ast}\ge {x}_{T-k-1}^{\ast} \) if and only if \( 1\ge {\delta}^{\frac{1}{1-\beta}}\left(\left(1+\gamma \right)-{\gamma \delta}^{\frac{1}{1-\beta}}\right) \), which is always true.∎

Proof of Proposition 8:

The proof proceeds by backward induction by showing that \( {W}_t\left({r}_t\right)=\sqrt{\left(1+{\sum}_{\tau =1}^{T-t}{\delta}^{-2\tau}\right)\ {r}_t\ \left(1-{r}_t\right)} \) for all t. To initialize the induction step, we have \( {W}_T\left({r}_T\right)={\max}_{{\overline{x}}_T\ge {r}_T\ge {\underline{x}}_T}\sqrt{\frac{r_T-{\underline{x}}_T}{{\overline{x}}_T-{\underline{x}}_T}{\left({\overline{x}}_T-{r}_T\right)}^2+\frac{{\overline{x}}_T-{r}_T}{{\overline{x}}_T-{\underline{x}}_T}{\left({r}_T-{\underline{x}}_T\right)}^2}=\sqrt{\left({\overline{x}}_T-{r}_T\right)\left({r}_T-{\underline{x}}_T\right)} \). Because the objective function is increasing in \( {\overline{x}}_T \) and decreasing in \( {\underline{x}}_T \), it is optimal to set \( {\overline{x}}_T=1 \) and \( {\underline{x}}_T=0 \). Hence, \( {W}_T\left({r}_T\right)=\sqrt{\ {r}_T\ \left(1-{r}_T\right)} \). Fix t < T and suppose that \( {W}_{t+1}\left({r}_{t+1}\right)=\sqrt{\left(1+{\sum}_{\tau =1}^{T-t-1}{\delta}^{-2\tau}\right)\ {r}_{t+1}\ \left(1-{r}_{t+1}\right)} \). In that case,

$$ {\displaystyle \begin{array}{c}{W}_t\left({r}_t\right)={\max}_{{\overline{x}}_t\ge {r}_t\ge {\underline{x}}_t}\sqrt{\frac{r_t-{\underline{x}}_t}{{\overline{x}}_t-{\underline{x}}_t}{\left({\overline{x}}_t-{r}_t\right)}^2+\frac{{\overline{x}}_t-{r}_t}{{\overline{x}}_t-{\underline{x}}_t}{\left({r}_t-{\underline{x}}_t\right)}^2}\\ {}\kern2em +{\delta}^{-1}\frac{r_t-{\underline{x}}_t}{{\overline{x}}_t-{\underline{x}}_t}\ {W}_{t+1}\left({\overline{x}}_t\right)+{\delta}^{-1}\frac{{\overline{x}}_t-{r}_t}{{\overline{x}}_t-{\underline{x}}_t}{W}_{t+1}\left({\underline{x}}_t\right)\\ {}\kern2em ={\max}_{{\overline{x}}_t\ge {r}_t\ge {\underline{x}}_t}\sqrt{\left({\overline{x}}_t-{r}_t\right)\left({r}_t-{\underline{x}}_t\right)}\\ {}\kern2em +\frac{r_t-{\underline{x}}_t}{{\overline{x}}_t-{\underline{x}}_t}\ \sqrt{\left(\sum \limits_{\tau =1}^{T-t}{\delta}^{-2\tau}\right)\ {\overline{x}}_t\ \left(1-{\overline{x}}_t\right)}\\ {}\kern2em +\frac{{\overline{x}}_t-{r}_t}{{\overline{x}}_t-{\underline{x}}_t}\sqrt{\left(\sum \limits_{\tau =1}^{T-t}{\delta}^{-2\tau}\right)\ {\underline{x}}_t\ \left(1-{\underline{x}}_t\right)}.\end{array}} $$

Taking the first-order optimality conditions yields that \( {\overline{x}}_t=\frac{1}{2}+\sqrt{{\left({r}_t-\frac{1}{2}\right)}^2+\frac{1}{1+\left({\sum}_{\tau =1}^{T-t}{\delta}^{-2\tau}\right)}{r}_t\left(1-{r}_t\right)} \) and \( {\underline{x}}_t=\frac{1}{2}-\sqrt{{\left({r}_t-\frac{1}{2}\right)}^2+\frac{1}{1+\left({\sum}_{\tau =1}^{T-t}{\delta}^{-2\tau}\right)}{r}_t\left(1-{r}_t\right)}. \)Substituting these values into the objective function yields:

$$ {\displaystyle \begin{array}{l}{W}_t\left({r}_t\right)\\ {}=\sqrt{\frac{1}{1+\left({\sum}_{\tau =1}^{T-t}{\delta}^{-2\tau}\right)}{r}_t\left(1-{r}_t\right)}\\ {}+\sqrt{\left(\sum \limits_{\tau =1}^{T-t}{\delta}^{-2\tau}\right)\ \left(\frac{1}{4}-{\left({r}_t-\frac{1}{2}\right)}^2-\frac{1}{1+\left({\sum}_{\tau =1}^{T-t}{\delta}^{-2\tau}\right)}{r}_t\left(1-{r}_t\right)\right)}\\ {}=\sqrt{\left(1+\left(\sum \limits_{\tau =1}^{T-t}{\delta}^{-2\tau}\right)\right){r}_t\left(1-{r}_t\right)},\end{array}} $$

completing the induction step. To see that this solution is indeed optimal, note that Wt(rt) corresponds to the optimal solution of the problem of allocation a total variance rt(1 − rt) across the T − t + 1 remaining periods so as to maximize \( {\sum}_{\tau =t}^T{\delta}^{t-\tau}\ \sqrt{v_{\tau}} \) subject to \( {\sum}_{\tau =t}^T{v}_{\tau}\le {r}_t\left(1-{r}_t\right) \) (Ely et al. 2015, online appendix). ∎

Proof of Proposition 9:

The proof proceeds by backward induction. In period T, \( {W}_T\left({r}_T\right)={\max}_{{\overline{x}}_T\ge {r}_T\ge {\underline{x}}_T}\frac{r_T-{\underline{x}}_T}{{\overline{x}}_T-{\underline{x}}_T}\ \left({\overline{x}}_T-{r}_T\right)+\frac{{\overline{x}}_T-{r}_T}{{\overline{x}}_T-{\underline{x}}_T}\left({r}_T-{\underline{x}}_T\right) \). Because the objective function is increasing in \( {\overline{x}}_T \) and decreasing in \( {\underline{x}}_T \), it is optimal to set \( {\overline{x}}_T=1 \) and \( {\underline{x}}_T=0 \). Hence, WT(rT) = 2rT(1 − rT). Therefore, in period T − 1,

$$ {\displaystyle \begin{array}{c}{W}_{T-1}\left({r}_{T-1}\right)={\max}_{{\overline{x}}_{T-1}\ge {r}_{T-1}\ge {\underline{x}}_{T-1}}\frac{r_{T-1}-{\underline{x}}_{T-1}}{{\overline{x}}_{T-1}-{\underline{x}}_{T-1}}\ \left({\overline{x}}_{T-1}-{r}_{T-1}\right)\\ {}+\frac{{\overline{x}}_{T-1}-{r}_{T-1}}{{\overline{x}}_{T-1}-{\underline{x}}_{T-1}}\left({r}_{T-1}-{\underline{x}}_{T-1}\right)\\ {}+{\delta}^{-1}\frac{r_{T-1}-{\underline{x}}_{T-1}}{{\overline{x}}_{T-1}-{\underline{x}}_{T-1}}\ {W}_T\left({\overline{x}}_{T-1}\right)\\ {}+{\delta}^{-1}\frac{{\overline{x}}_{T-1}-{r}_{T-1}}{{\overline{x}}_{T-1}-{\underline{x}}_{T-1}}{W}_T\left({\underline{x}}_{T-1}\right)\\ {}={\max}_{{\overline{x}}_{T-1}\ge {r}_{T-1}\ge {\underline{x}}_{T-1}}2\ \frac{r_{T-1}-{\underline{x}}_{T-1}}{{\overline{x}}_{T-1}-{\underline{x}}_{T-1}}\ \left({\overline{x}}_{T-1}-{r}_{T\_1}\right)\\ {}+2\ {\delta}^{-1}\Big(\frac{r_{T-1}-{\underline{x}}_{T-1}}{{\overline{x}}_{T-1}-{\underline{x}}_{T-1}}{\overline{x}}_{T-1}\ \left(1-{\overline{x}}_{T-1}\right)\\ {}+\frac{{\overline{x}}_{T-1}-{r}_{T-1}}{{\overline{x}}_{T-1}-{\underline{x}}_{T-1}}{\underline{x}}_{T-1}\left(1-{\underline{x}}_{T-1}\right)\Big).\end{array}} $$

Taking the first-order optimality conditions with respect to \( {\underline{x}}_{T-1} \) and \( {\overline{x}}_{T-1} \) (and ignoring the suboptimal non-informative solutions) yields the following solution: \( {\underline{x}}_{T-1}={r}_{T-1}-\delta /4 \) and \( {\overline{x}}_{T-1}={r}_{T-1}+\delta /4 \) if \( {r}_{T-1}\in \left[\frac{\delta}{4},1-\frac{\delta}{4}\right] \), \( {\underline{x}}_{T-1}=0 \) and \( {\overline{x}}_{T-1}=\sqrt{\delta\ {r}_{T-1}} \) if \( {r}_{T-1}\in \left[0,\frac{\delta}{4}\right] \), and \( {\underline{x}}_{T-1}=1-\sqrt{\delta\ \left(1-{r}_{T-1}\right)} \) and \( {\overline{x}}_{T-1}=1 \) if \( {r}_{T-1}\in \left[1-\frac{\delta}{4},1\right]. \) Hence,

$$ {\displaystyle \begin{array}{l}{W}_{T-1}\left({r}_{T-1}\right)\\ {}=\left\{\begin{array}{cc}2{r}_{T-1}\left(1+\delta \right)-4{r}_{T-1}\sqrt{\delta\ {r}_{T-1}}& \mathrm{if}\ {r}_{T-1}\in \left[0,\frac{\delta}{4}\right]\\ {}\kern0.5em {\delta}^{-1}2{r}_{T-1}\left(1-{r}_{T-1}\right)+\frac{\delta}{8}& \mathrm{if}\ {r}_{T-1}\in \left[\frac{\delta}{4},1-\frac{\delta}{4}\right]\\ {}2\left(1-{r}_{T-1}\right)\left(1+\delta \right)-4\left(1-{r}_{T-1}\right)\sqrt{\delta\ \left(1-{r}_{T-1}\right)}& \mathrm{if}\ {r}_{T-1}\in \left[1-\frac{\delta}{4},1\right].\end{array}\right.\end{array}} $$

Consider next period T − 2:

$$ {\displaystyle \begin{array}{c}{W}_{T-2}\left({r}_{T-2}\right)={\max}_{{\overline{x}}_{T-2}\ge {r}_{T-2}\ge {\underline{x}}_{T-2}}\frac{r_{T-2}-{\underline{x}}_{T-2}}{{\overline{x}}_{T-2}-{\underline{x}}_{T-2}}\ \left({\overline{x}}_{T-2}-{r}_{T-2}\right)\\ {}+\frac{{\overline{x}}_{T-2}-{r}_{T-2}}{{\overline{x}}_{T-2}-{\underline{x}}_{T-2}}\left({r}_{T-2}-{\underline{x}}_{T-2}\right)\\ {}+{\delta}^{-1}\frac{r_{T-2}-{\underline{x}}_{T-2}}{{\overline{x}}_{T-2}-{\underline{x}}_{T-2}}\ {W}_{T-1}\left({\overline{x}}_{T-2}\right)\\ {}+{\delta}^{-1}\frac{{\overline{x}}_{T-2}-{r}_{T-2}}{{\overline{x}}_{T-2}-{\underline{x}}_{T-2}}{W}_{T-1}\left({\underline{x}}_{T-2}\right).\end{array}} $$

When \( {r}_{T-2}\in \left[\frac{\delta +{\delta}^2}{4},1-\frac{\delta +{\delta}^2}{4}\right] \), the function is maximized at \( {\underline{x}}_{T-2}={r}_{T-2}-{\delta}^2/4 \) and \( {\overline{x}}_{T-2}={r}_{T-2}+{\delta}^2/4 \). ∎

Rights and permissions

Reprints and permissions

Copyright information

© 2019 Springer Nature Switzerland AG

About this chapter

Check for updates. Verify currency and authenticity via CrossMark

Cite this chapter

Roels, G. (2019). Optimal Structure of Experiential Services: Review and Extensions. In: Maglio, P.P., Kieliszewski, C.A., Spohrer, J.C., Lyons, K., Patrício, L., Sawatani, Y. (eds) Handbook of Service Science, Volume II. Service Science: Research and Innovations in the Service Economy. Springer, Cham. https://doi.org/10.1007/978-3-319-98512-1_6

Download citation

Publish with us

Policies and ethics